LoginSignup
0
0

More than 5 years have passed since last update.

Excelの統計分布系の関数の話 1(ポワソン分布編)

Last updated at Posted at 2018-12-27

Excelの統計分布系の関数の話 1

Excelを使っていて、これまで集計したデータを元に予測分析したくなることがあります。
しかし、統計に関して出回っている書籍の多くが「標準偏差」から「回帰分析」あたりの関数までを主な範囲としていて、「統計」カテゴリの関数をあまり使う機会はありませんでした。

今回は、自分自身のデータの分析と予測で必要となった「ポワソン分布」を初めとして、以下の簡単な解説を掲載します。

免責

  • 初版は、公式との対応、もしくはパラメータの解釈のみです
  • 近似できる部分などの「証明」については、追って掲載します
  • 数学としての正確な書き方はしておりません

取り扱う統計関数

  • ポワソン分布 (POISSON.DIST)
  • ガンマ分布 (GAMMA.DIST)
  • 負の二項分布(NEGBINOM.DIST)

基本的に~.DISTとつくものは、結果の値は $0 \leq y \leq 1$ の小数値で返され、書式を%に合わせると良いです。

サンプルデータ

a b
2 0
3 1
2 1
2 0
3 1
1 1

後で使う値 $\lambda$ は以下の通りとします。

\begin{align}
\lambda &= \sum_{k=1}^{n} \frac{b_k}{a_k} = 0.308...\\
\Leftrightarrow \lambda^{-1} &= 3.25
\end{align}

ポワソン分布編

ポワソン分布は、まれに発生する事象(1週間に発生する交通事故など)に適用出来る確率分布です。
単位時間(または事象1回の)の発生確率 $\lambda$ に対し、 $x$ 件発生する確率を求められます。

数式

グラフは直接はご用意できませんが、カシオの計算サイトが分かりやすいのでご覧下さい。
ポアソン分布(グラフ)

ポワソン分布は、$x \in \mathbb{N}$ すなわち $x = 0, 1, 2, 3, ...$ の自然数(諸説あり)と、xの値1個(単位)ごとの平均 $\lambda$ をパラメータにとります。数式は以下の通り。

f(x, \lambda) = \frac{e^{-\lambda}\lambda^x}{g(x)}, \quad g(x) = 
\left\{
\begin{array}{ll}
x \in \mathbb{Z} &\Rightarrow \Gamma(x+1) \\
x \in \mathbb{N} &\Rightarrow x!
\end{array}
\right.

以上の式が、POISSON.DIST(x,λ,FALSE) に置き換えられます。

補足

  • $x = 0$ のとき、 $0! = \Gamma(1) = 1$ です。
  • $x = 1$ のとき、指数分布 $f(x) = \lambda e^{-\lambda x}$ と一致します。
  • $g(x)$ の上段はガンマ関数 $\Gamma(x)$ といい、階乗の取る値を正の実数へ拡張したものです。
  • ガンマ関数は、 $x \in \mathbb{N}$ なら $\Gamma(x+1) = x!$ で置き換えられます。
  • Excelは下段の自然数の場合を採用しています。

なお、ガンマ関数の定義は以下の通りです。

\Gamma(s) = \int_{0}^{\infty}x^{s-1}e^{-x}dx

Excelの関数使用時の注意(エラーとなる範囲)

  • $x \in \mathbb{Z}$ の場合、小数点以下は切り捨てられ $x \in \mathbb{N}$ と扱われます。(実数としての実装は自力のみ)
  • 数値以外はエラーになります。
  • また、$x < 0, \lambda < 0$ の場合もエラーとなります。

※以下は検証中のため、正確さは保証できません

  • 条件は $x \geq 0$ , $0 < \lambda \leq 6$ あたりが望ましそうです。(再度検証の可能性あり)

累積確率

関数を使って $0 \leq k \leq x$ で累積確率を取りたい場合、Excelの場合は $x \in \mathbb{N}$ を取るので、Σを使います。
このときの関数を下記の式に対応させると、 POISSON.DIST(x,λ,TRUE) です。

\sum_{k=0}^{x} f(k,\lambda) = e^{-\lambda}\sum_{k=0}^{x} \frac{\lambda^k}{k!} \quad (0! = 1)

例題(追記予定)

問題は用意してあるのですが、後日追記します。

0
0
0

Register as a new user and use Qiita more conveniently

  1. You get articles that match your needs
  2. You can efficiently read back useful information
  3. You can use dark theme
What you can do with signing up
0
0